GMAT Ninja Quant Ep 1: Arithmetic

Поділитися
Вставка
  • Опубліковано 11 січ 2025

КОМЕНТАРІ • 69

  • @keerthana6661
    @keerthana6661 2 місяці тому +22

    Great set of questions! Must say that the methods used, boosts the confidence too.
    However, would like to suggest an alternative to the Question 3: Approximate to 0.7, 0.3 and 0.7 respectively. Calculate the product 0.7/(0.3*0.7). Cancel out 0.7s so 1/0.3 becomes 10/3 ~ 3.33. Hence the answer D.

  • @lucasettanni3301
    @lucasettanni3301 2 місяці тому +6

    You guys are the best! Thanks for providing such helpful content for free, really appreciated

  • @MariaFelipe-q5p
    @MariaFelipe-q5p 3 місяці тому +10

    Thank you for making this course so accessible, really appreciate it

    • @GMATNinjaTutoring
      @GMATNinjaTutoring  2 місяці тому +4

      Thank you for watching! I'm glad that it's helping a bit. Have fun studying!

  • @adityap4222
    @adityap4222 3 місяці тому +3

    Here's my approach for question 4 (27:21), instead of working with the numerator it would be easier to "stretch" the denominator to get exactly 1/2, here's how it would work in practice;
    A. 9/16 --> stretch/increase denominator by 2 (9/18) to make it 1/2
    B. 10/19 --> stretch/increase denominator by 1 (10/20) to make it 1/2
    C. 13/24 --> stretch/increase denominator by 2 (13/26) to make it 1/2
    D. 16/30 --> stretch/increase denominator by 2 (16/32) to make it 1/2
    E. 19/36 --> stretch/increase denominator by 2 (19/38) to make it 1/2
    You'll notice that option B requires the least stretch/increase and therefore it would be the closest to 1/2, I believe this helps to analyse and conclude quickly, what are your thoughts?

    • @GMATNinjaTutoring
      @GMATNinjaTutoring  3 місяці тому +4

      I like your thinking, thank you for posting this method!
      I'd be reluctant to use it in a question like this, however, because of how 'stretching' the denominator by the same number would mean a different sized change, depending on what the fraction is.
      For example, stretching the denominator of 50/94 by 6 would involve a smaller change than stretching the denominator of 7/13 by 1, even though a 'stretch' of 6 looks like a much bigger change than a 'stretch' of 1.
      In this question, the denominators are not that far apart, so the 'stretch' method works here, but I wouldn't like to have to decide when it will or will not work under the time pressure of the exam. While the method shown in the video will likely take more time, it's a little safer than using this method.
      Thank you for your comment, and good luck with your GMAT studies!

    • @adityap4222
      @adityap4222 3 місяці тому +3

      @@GMATNinjaTutoring That makes total sense, and you're right about my method would not hold true if the denominators are spread apart, thanks for pointing out :)

  • @siddharthkandpal9341
    @siddharthkandpal9341 2 місяці тому +5

    Another easier way for q2 could be -->
    Since the amount adds upto 10k, hence we can write the avg calculation as -> 50(0.4)+100(0.3)+150(0.25)+250(0.05).
    Taking 50 out, 50(0.4+0.6+.75+.25)= 50X(1+1)=100

  • @deobitales3912
    @deobitales3912 2 місяці тому

    Thank you! as someone who hasn't touched maths since 12th grade (been over 6 years) and a working professional... this really really simplified it and omg! thank you for making it accessible! appreciate it a great deal

  • @masterhind2036
    @masterhind2036 Місяць тому +1

    In question 3, if you could round off both 0.7197 and 0.739 to 0.7, you'll be left with 1/0.3 which would give us 3.33 which is between 3 and 4.

  • @yashlikhar831
    @yashlikhar831 3 місяці тому +1

    Amazing course great value

  • @shreyway21
    @shreyway21 3 місяці тому +6

    Hello. at 18:56 you times all the variables by 10, which makes sense. but in step 3, you didn't multiply 0.74 by 10, you just rewrote it as the same vale (0.74) . That looks wrong. could you clarify this?

    • @GMATNinjaTutoring
      @GMATNinjaTutoring  3 місяці тому +4

      It's not that Harry multiplied all the variables by 10, it's that he multiplied the top and bottom of the fraction by 10.
      If we multiplied all three variables by 10, we'd multiply the top of the fraction by 10 and the bottom of the fraction by 100. This would change the value of the fraction, and we can't do that.
      Instead, he multiplied the top and bottom of the fraction by 10, which we could represent at 10/10. This doesn't change the value of the overall fraction which is why it's ok to do this.
      I hope that helps!

    • @bikelockmusic
      @bikelockmusic 3 місяці тому +3

      @@GMATNinjaTutoringStill confused by this too 😂

    • @Superawesomeclonewar
      @Superawesomeclonewar 3 місяці тому +2

      @@bikelockmusic Think of the inequality as if it's P = [0.72/0.294 x 1/0.739]. When he multiplies by 10/10 it would equal 7.2/2.94 x 1/0.739. He then rounds up the 2.9 to 3 to simplify the 7.2. You then get 2.4 x 1/0.74 which you can then multiply by 100/100 to get 240/74. Hope that helps!

    • @sotonwasuccess3011
      @sotonwasuccess3011 16 днів тому

      @@Superawesomeclonewarwhere did the 2.4 come from?

  • @sajalbatwara5342
    @sajalbatwara5342 18 днів тому

    You guys are best! :)

  • @mili-d7c4h
    @mili-d7c4h 29 днів тому

    Loved this, Thank You !

  • @hosseinj8149
    @hosseinj8149 Місяць тому +1

    I would really appreciate a timer during pauses for the questions! That way you guys get more watch time instead of us just pausing, and it will simulate a time crunch for us!

  • @OpenCSE
    @OpenCSE 4 місяці тому +1

    Thank You Harry 🎉

  • @tanyavarshney8638
    @tanyavarshney8638 3 місяці тому +2

    although i was learning and listeneing, couldnt unsee that u do have bright blue eyes

    • @GMATNinjaTutoring
      @GMATNinjaTutoring  3 місяці тому +2

      Now you know our secret: we hired Harry mostly because of his beautiful eyes. 😆

    • @anindyatrehan7871
      @anindyatrehan7871 3 місяці тому +1

      @@GMATNinjaTutoring Harry Potter

  • @thehappygoluckygirll.9
    @thehappygoluckygirll.9 3 місяці тому +2

    In 3rd question, I was doing evrything right, just by the end while calculating 120/37...I took approx 36 to make it easier and got E as answer. But the answer was D ofcourse! :(

    • @siddharthkandpal9341
      @siddharthkandpal9341 2 місяці тому

      120/36 = 30/9 = 3.something, still D. Maybe you miscalculated. Nothing wrong with approximating to 36 ig.

  • @DS-pg6wh
    @DS-pg6wh Місяць тому

    Great Video! Thank you very much. That really helps :)

  • @diya772364
    @diya772364 2 місяці тому

    For Q4, just simply subtract 1/2 from each number and all will be of the form mentioned above. The one with the highest denominator is the closest.

    • @manisharaste4918
      @manisharaste4918 Місяць тому

      Hi, can you share some tips for preparing quant?

  • @achitodua3695
    @achitodua3695 26 днів тому

    In the last question, we can estimate the prices h and p to 6 and 0.8, then write
    6h + 0.8p > 700,
    60h +8p>7000,
    52h + 8*(h+p) > 7000,
    52h + 8*120 > 7000,
    52h > 6040,
    this means h should be well over 100 to get more than 7000 profit, and first question does not give us enough info to know that, while second does.

    • @mathc.5981
      @mathc.5981 20 днів тому

      Hey, can you please explain "52h + 8*(h+p) > 7000?"

  • @ShivMAUS
    @ShivMAUS 3 місяці тому +2

    Hi, regarding the last question, the question stem asks "was bo's profit more than 700 yesterday", implying we are only trying to answer a 'yes' or 'no' rather than asking us to calculate a specific value. I have been studying with TTP and it made a distinction of data sufficiency questions either being Yes/No or Value questions and this one seems to fall into Yes/No.
    Meaning statement 1 should be sufficient as it allows us to derive a value which either tells us "yes" or "no" to the question stem (was the profit over or under 700) and the information in statement 1 enables us to give an answer thus making it sufficient. Likewise with statement two, this would make it sufficient as well right?
    I have only just started prepping for the GMAT so my understanding may be missing some key facts, so happy to be corrected here. Thanks so much for your vids!

    • @GMATNinjaTutoring
      @GMATNinjaTutoring  3 місяці тому +4

      You're absolutely right that this is a Yes/No question, but statement 1 is insufficient to answer this question.
      From the information in statement 1, we can create one scenario in which Bo's profit is greater than $700 and one scenario in which Bo's profit is less than $700. This means that we can say both Yes AND No from the information in statement 1, meaning statement 1 provides insufficient information to answer this question.
      From the information in statement 2, we can only create scenarios in which Bo's profit is less than $700. This means we can only say No to the question, meaning statement 2 provides sufficient information to answer the question.
      It's a lot of work to create these scenarios. That's why Harry focused on estimation (either under or over estimation) and testing the extreme scenarios suggested by the information in each statement to reduce the amount of work required by this problem.
      I hope that helps, but please feel free to ask any other questions!

    • @ShivMAUS
      @ShivMAUS 3 місяці тому +1

      @@GMATNinjaTutoring Thanks so much for your reply, so to confirm, for these yes/no categories of questions, even if we can deduce from the first statement that the answer is "no" then that would mean it is always insufficient. meaning for these yes/no type questions, the sufficiency only depends on giving a "yes" answer to the question.
      Are there cases for yes/no questions where a statement woud still be considered sufficient even if it gives us a "no" answer?

    • @mehul6687
      @mehul6687 3 місяці тому +3

      @@ShivMAUS No, thats not right. In statement 2, we can come to the conclusion that the answer is no so statement 2 is sufficient for us to answer this question as its a yes or no question. However, in statement 1, the answers we can come up with are inconclusive as it can be both yes or no, so it's not sufficient.
      In yes/no type questions, if you can come to a conclusive answer, whether thats yes or not, that statement would be sufficient. But if the answer is inconclusive or somehow we can derive both yes and no from it, it is not sufficient. Hope that helps!

  • @DolphRehbein
    @DolphRehbein 3 місяці тому

    43:20 I don't understand how this helps you hedge your bets. If having h=1 made the difference and pushed the sum over $7.00 you would have said statement 1 is sufficient, continued to statement 2 and have gotten the problem wrong, because no where did it state h>0. Just before that at 41:30, I have trouble with these estimations and where to draw the line between making the numbers easier to calculate and capturing enough info to make the estimation useful. I estimated $6.00 *100 + $0.80*100 which came in below $700 and realized it was close and had to do the long multiplication to check that it was actually was >$700. Do you have any tips for doing how to approach this trade off on the GMAT? When I see solutions like this I feel that the estimation was made with prior knowledge of the actual solution.

    • @joon259
      @joon259 3 місяці тому

      He's trying to get the total value over $700 when evaluating h=1 and p=213, because if that were the case then A would be sufficient. The plates of hummus are where the majority of the profit will be derived, and he proved that Statement 1 can prove to be >$700 when h=106 and p=108. So he's looking for a way to prove that Statement 1 will be sufficient for all cases of h and p, considering the case where the total sum will be at its lowest with h=1. If he's able to overestimate given the lowest possible profit and still have the sum be less than $700, then you know that the actual value is guaranteed to be under as well. Not sure if this helps at all.

    • @GMATNinjaTutoring
      @GMATNinjaTutoring  3 місяці тому +2

      Let me take your comments one at a time, and I'll see if I can help at all.
      At 43:20, the choice between h=0 and h=1 didn't make a difference to the outcome of the question in the end. However, it's a good idea to be thinking about whether we could have a scenario in which zero plates of hummus were sold. Considering things like this makes it more likely we'll pick up on the clues in the text of another question if it did say something to the effect of "Bo made at least one sale of each type of food."
      Your question about how to estimate the numbers is a great one that I don't have a perfect answer for. A lot of it comes down to your comfort with numbers and experience with these sorts of questions. In the calculation you listed, you've underestimated three of the values and overestimated (even if only slightly) one of the values. I'd recommend you either make overestimations or underestimations, depending on what you're trying to show. At this point, Harry was aiming to show that Bo could have made a profit of over $700, using the information available. If he underestimated some of the values and the figure came out over $700, we'd definitely know the real figure was over $700.
      If we were to do $6.00*100 + $0.79*100, we'd get 600 + 79 = 679. Perhaps instead of doing the long multiplication, we could think of ways of making this calculation more accurate without adding too much to our workload. I'd suggest the easiest thing we could do here is to say we have 106 people eating hummus rather than 100 so our calculation would be $6.00*106 + $0.79*100. This would add $36 to our total giving Bo an estimated profit of $715. We've still underestimated the profit of one plate of hummus and the number of people buying packs of pitta, so the real value is still higher than $715, but we've shown that in a way that doesn't involve too much additional calculation.
      In the initial estimation, there's a trade-off between making the calculation easier and getting as close as possible to the real figure. However, we can always adjust the estimation to get closer to the real value, as shown in the paragraph above. This will never be an exact science, but you'll hopefully get more comfortable with this sort of adjustment with some practice.
      I hope that helps a bit, but please let me know if you have any other questions!

  • @rishabhgoenka9686
    @rishabhgoenka9686 2 місяці тому +3

    For Q3, I rounded to 0.7/0.3*0.7 = 1/0.3 = 10/3 = 3.333
    It gives the right answer for this question but would you say that is rounded by too much and may not work for other questions similar to that one?

  • @mwitiraaugustine1098
    @mwitiraaugustine1098 3 місяці тому +1

    Thank you Sir!

  • @SanjukthaDevanathan
    @SanjukthaDevanathan 25 днів тому

    For Q 4 .. I reversed the num and den for all fractions and simplified to see which one would be the closest to 2( 1/2 after the num and den are switched)
    Only 19 divided by ten was 1.9. So I chose that option.
    I am not sure if there is any logic behind this process. Please help me understand if the answer was right by fluke..

  • @guillaumegruyters2471
    @guillaumegruyters2471 3 місяці тому +1

    I'm confused about the final question, 1, is insufficient because the answer could possibly be profitable and unprofitable. and 2. is simply just wrong? Like there is one answer, no either or?
    Also in Q3, why did you only multiply 0.3 to 3 and not 0.74 to 7.4 automatically?
    Thank you!

    • @GMATNinjaTutoring
      @GMATNinjaTutoring  3 місяці тому +2

      In the final question, the information in statement 2 tells us that Bo definitely made a profit that was less than $700. We don't know exactly how much less, because we overestimated the amount of profit he made for each item, but we know he made less than $592 so he definitely made less than $700. This means we can answer the question of whether he made more than $700 profit yesterday using the information from statement 2 alone, so (B) is the answer to this question.
      In Q3, we're changing how a fraction looks by multiplying the top and bottom of that fraction by the same number. This is a similar process to when we want to create a common denominator when we're adding fractions. If we have 0.72/(0.3*0.74) and we multiply the top and bottom of the fraction by 10, we'd have (10*0.72)/(10*0.3*0.74). We could then use that 10 to give 7.2/(3*0.74). We could also have multiplied the 0.74 by 10 instead to give 7.2/(0.3*7.4).
      However, we can't multiply BOTH 0.3 and 0.74 by 10 because then we'd have multiplied the top of the fraction by 10 and the bottom of the fraction by 10 twice, so we'd have multiplied the bottom by 100. This would change the value of the fraction and we wouldn't get the right answer.
      I hope that helps!

  • @ThiKimTuHuynh-h7x
    @ThiKimTuHuynh-h7x 5 днів тому

    For question 6, I don't understand why B is the answer, because if your hypothesis h=63, p=151 and total

    • @GMATNinjaTutoring
      @GMATNinjaTutoring  5 днів тому +3

      In a Data Sufficiency question, we're trying to figure out whether we can answer the question. We're not really concerned about whether the answer is 'yes' or 'no', just whether we can tell one way or the other. In this question, the second statement is sufficient for us to say 'no' to the question -- we know that Bo made less than $700 profit yesterday. This means this information is sufficient to answer the question, so (B) is the correct answer choice.
      I hope that helps!

  • @lesdickson9765
    @lesdickson9765 3 місяці тому +1

    I'm prepping for GMAT and going through these questions but I've found myself not simplifying my process for Q4 & Q5 -- it took me about 3:30-4 mins to answer both questions despite getting them right which will hold me back in the real (or mock) exams. How do I simplify my process and not rely so much on calculations?
    edit: for Q6, I worked it out differently by using the actual numbers without over or underestimating either quantity or price and got around $485 but got the question as I chose E instead of B smh. I did get Q1-Q5 right though, the last one was a challenge but at least I'm starting to internalise the processes to answer such questions.

  • @mizrahijean6168
    @mizrahijean6168 Місяць тому

    Bonjour,
    Concernant la question 5, j'ai eu une différente approche. En regardant les réponses, j'ai vu qu'elle comprennais toutes une égalité. Donc 2 des 4 étaient forcement égaux. Il m'a suffit de voir lesquels étaient potentiellement égaux (vu que deux étaient forcement égaux s'il semblent égaux, il le sont) sans prendre en compte les 0 et prendre la réponse qui présente cette relation.
    Cette technique ne fonctionne peut être uniquement dans cette situation mais elle m'a donné la réponse en 3 secondes.

  • @doniaridane175
    @doniaridane175 3 місяці тому +1

    aren't we allowed to use calculators?

    • @GMATNinjaTutoring
      @GMATNinjaTutoring  3 місяці тому

      You can use a calculator in the Data Insights section of the GMAT, but you can't use a calculator in the Quant section.
      I hope that helps!

  • @KritikaDudeja-e8y
    @KritikaDudeja-e8y 2 місяці тому

    how did you come to 108 and 106 in the last question?

    • @GMATNinjaTutoring
      @GMATNinjaTutoring  2 місяці тому

      At that point, Harry was demonstrating how much profit Bo would make if he sold the least number of pittas possible, given the restriction provided by the information in statement 1.
      If Bo sold a total of 214 packs of pitta and plates of hummus, so p + h = 214, and he sold more packs of pitta than plates of hummus, so p > h, then the smallest value p can take is 108. If p = 108, then h = 106.
      I hope that helps!

  • @Dokyi123
    @Dokyi123 2 місяці тому

    Can someone explain why 0.74 was left unchanged in step 3 of Q3 and also why the 3 was left unchanged in step 4?

    • @GMATNinjaTutoring
      @GMATNinjaTutoring  2 місяці тому

      In step 3, Harry multiplied the fraction by 10/10 and in step 4, he multiplied the fraction by 100/100. This is the same process as when we want to add two fractions together and need to find a common denominator.
      In step 3, he multiplied the whole numerator and denominator by 10. He did not multiply each individual term by 10. You could think of the change in the denominator as (10)(0.3)(0.74) = (3)(0.74).
      Similarly, you could think of the change to the denominator in step 4 as (3)(0.74)(100) = (3)(74).
      I hope that helps, but please let me know if you have any other questions!

    • @jellypush1234
      @jellypush1234 2 місяці тому

      @@GMATNinjaTutoring So when you multiplying the whole numerator and whole denominator, only one of the numbers in the term gets multiplied?

    • @GMATNinjaTutoring
      @GMATNinjaTutoring  2 місяці тому +1

      Yes. It might be easier to see this if we did this with a "normal" multiplication instead of using a fraction. If we had 5*3 and wanted to multiply this by 10 we'd get 10*5*3 which we could rewrite as 50*3 or 5*30. What we can't do is say that that 10*5*3 = 50*30.
      What Harry did in this question was the same as what's written above, but he did it to the top and bottom of the fraction at the same time.
      I hope that helps!

  • @beckyilelanwo9054
    @beckyilelanwo9054 3 місяці тому +1

    C how

  • @AryanJain-o3l
    @AryanJain-o3l 3 дні тому

    if gmat is looking for donating money to charity they can approach me i am the most poorest boy over here

  • @deannagarcia327
    @deannagarcia327 13 днів тому

    This presentation was delivered with too much nervousness. He stressed me out. Presenter was constantly in the way of reading the dry erase board. I did not enjoy it.

  • @nagatopain8453
    @nagatopain8453 4 місяці тому +9

    Hey, look at his eyes, pure blue

    • @BASIT_BRO
      @BASIT_BRO 2 місяці тому

      Nice observation gmater

  • @SwethaReddyLakshmanan
    @SwethaReddyLakshmanan Місяць тому

    I'm sorry, but while solving Q2- I calculated only the mean of the price of ticket listed and marked 125 as the answer. Now after seeing Harry solving it and going back again & reading the question, I still am not seeing why we have to take the no.of seats too? Please help me see this! I'm lost here🥲

    • @GMATNinjaTutoring
      @GMATNinjaTutoring  Місяць тому +1

      To find the average price of a seat, we need to find the total cost of all the seats and then divide by the number of seats. We can't do this unless we use the number of seats in our solution. Let's change the question to see if looking at things slightly differently helps you see why we need to take the number of seats into account. Consider the case of a theatre that only has 10 seats. For 9 of the seats, it costs $10 to see a show but for the final seat, it costs $20. If all we needed to pay attention to was the cost, we'd find the average by doing (10 + 20) / 2 = 15. However, the real average is (10 + 10 + 10 + 10 + 10 + 10 + 10 + 10 + 10 + 20) / 10 = 11.
      The solution to question 2 uses the same principle outlined above, but the numbers are bigger. Instead of adding up a lot of 50s and a lot of 100s, we can use multiplication to make the equation simpler. That's how Harry ended up with (4000*50 + 3000*100... and so on.
      I hope that helps!

    • @SwethaReddyLakshmanan
      @SwethaReddyLakshmanan Місяць тому

      @@GMATNinjaTutoring Now i get it! Thank you so much for the great explanation. Have a beautiful day💛